If you find any mistakes, please make a comment! Thank you.

Solution to Principles of Mathematical Analysis Chapter 6 Part A


Chapter 6 The Riemann-Stieltjes Integral

Exercise 1

(By Matt Frito Lundy) Note: I should probably consider the cases where $x \pm d \notin [a,b]$ in the solutions to 1 and 2 below. First note that for any partition $P$ of $[a,b]$, $L(P,f,\alpha) = 0$, so we have $$\underline{\int_a^b} f \ d\alpha = 0$$ and $0 \leq U(P,f,\alpha)$.

Let $\varepsilon > 0$ be given. $\alpha$ is continuous at $x_0$ means there exists a $\delta > 0$ such that $\left| x – x_0 \right| < \delta$ and $a \leq x \leq b$ implies $\left| \alpha(x) – \alpha(x_0) \right| < \varepsilon$.

Let $P = \{a, x_0 – \delta, x_0 + \delta, b\}$. Then we have
$$U(P,f,\alpha) = \alpha(x_0 + \delta) – \alpha(x_0 – \delta) < 2\varepsilon$$ and
$$0 \leq U(P,f,\alpha) < 2\varepsilon.$$ Because $\varepsilon$ was arbitrary, we have
$$\overline{\int_a^b} f \ d\alpha = 0$$ so that $f \in \mathscr{R}(\alpha)$ and
$$\int_a^b f \ d\alpha = 0.$$


Exercise 2

(By Matt Frito Lundy) Suppose there exists an $x \in [a,b]$ such that $f(x) > 0$. $f$ is continuous at on $[a,b]$ means there exists a $\delta$ such that $|t – x| < \delta$ and $a \leq t \leq b$ implies $f(t) > 0$. Let $P = \{a, x – \delta, x+ \delta, b\}$, then $L(P,f) > 0$, and for any partition $P$ $$\int_a^b f(x) \ dx \geq L(P,x) > 0,$$ a contradiction.


Exercise 3

(By Matt Frito Lundy)

(a) First suppose that $f(0+) = f(0)$ and let $\varepsilon > 0$ be given. Then there exists a $\delta^* >0$ such that $0 < x < \delta^*$ implies $\left| f(x) – f(0) \right| < \epsilon$. Let $\delta = \min\{1,\delta^*/2\}$ and form a partition $P = \{-1,0,\delta,1\}$. Then we have
$$U(P,f,\beta_1) – L(P,f,\beta_1) = f(s) – f(t)$$ for some $s,t \in [0,\delta]$. But
$$f(s) – f(t) \leq \left| f(s) – f(0) \right| + \left| f(0) – f(t) \right| $$ so we have
$$U(P,f,\beta_1) – L(P,f,\beta_1) \leq \left| f(s) – f(0) \right| + \left| f(0) – f(t) \right| < 2\varepsilon,$$ which shows that $f \in \mathscr R (\beta_1)$.

Now suppose that $f \in \mathscr R (\beta_1)$ and let $\varepsilon >0$ be given. Then there exists a partition $P$ for which $$U(P,f,\beta_1) – L(P,f,\beta_1) < \varepsilon.$$ Let $P^*$ be a refinement of $P$ that includes $0$. Then we have:
$$U(P^*,f,\beta_1) – L(P^*,f,\beta_1) < \varepsilon.$$ But if $[0,\delta]$ is the subinterval of $P^*$ that contains $0$, then:
$$U(P^*,f,\beta_1) – L(P^*,f,\beta_1) = f(s) – f(t)$$ for some $s,t\in[0,\delta]$ where $f(s) – f(t) \geq \left| f(x_1) – f(x_2) \right|$ for any $x_1,x_2 \in [0,\delta]$. So we have for any $0 < x < \delta$
$$\left| f(0) – f(x)\right| \leq f(s) – f(t) < \varepsilon,$$ which means that $f(0+) = f(0)$.

As above, for any partition $P$ that contains $0$ we have:
$$U(P,f,\beta_1) = M_l \quad L(P,f,\beta_1) = m_l$$ in the interval $[0,x_l]$ of $P$. Because $f$ is right-continuous at $0$, both $M_l$ and $m_l$ converge to $f(0)$ as $x_l \to 0$, so $$\int f \ d\beta_1 = f(0).$$

(b) The statement is: $f\in \mathscr R (\beta_2)$ if and only if $f(0-) = f(0)$ and then
$$\int f \ d\beta_2 = f(0).$$ The proof is similar to part (a).

(c) Suppose that $f$ is continuous at $0$ and let $\varepsilon$ be given. Then there exists a $\delta^* > 0$ such that $\left| x \right| < 0$ implies $\left| f(x) – f(0) \right| < \varepsilon$. Let $\delta = \min\{1,\delta^*/2\}$, and $P = \{-1,-\delta,\delta,1\}$. Then
$$U(P,f,\beta_3) – L(P,f,\beta_3) = f(s) – f(t)$$ where $s,t \in [-\delta,\delta]$. But
$$f(s) – f(t) \leq \left| f(s) – f(0) \right| + \left| f(0) – f(t) \right| $$ so
$$U(P,f,\beta_3) – L(P,f,\beta_3) < 2\varepsilon$$ and $f\in \mathscr R (\beta_3)$.

Now suppose that $f\in \mathscr R (\beta_3)$ and let $\varepsilon > 0$ be given. There exists a partition $P$ such that
$$U(P,f,\beta_3) – L(P,f,\beta_3) < \varepsilon.$$ Let $P^*$ be a refinement of $P$ that contains $0$ so that the partitions around $0$ are $[x_{l-1},0]$ and $[0,x_l]$, let $\delta = \min\{\left| x_l \right| ,\left| x_{l-1} \right| \}$ and let $P^\#$ be a refinement of $P^*$ that contains $\pm \delta$. Then
$$U(P^\#,f,\beta_3) – L(P^\#,f,\beta_3) = \frac{1}{2} \left[ f(s) – f(t) + f(q) – f(r)\right] $$ where $s,t \in [0,\delta]$, $q,r \in [-\delta,0]$ $$f(s) – f(t) \geq \left| f(0) – f(x) \right|
\quad \text{for any} \quad x \in [0,\delta]$$ $$f(q) – f(q) \geq \left| f(0) – f(x) \right|
\quad \text{for any} \quad x \in [-\delta, 0].$$ So for any $x \in [-\delta,\delta]$ we have
$$\left| f(0) – f(x) \right| < \epsilon$$ which shows that $f$ is continuous at $0$.

(d) The result follows from parts (a) – (c) and the fact that if $f$ is continuous at $0$, then $f(0-) = f(0) = f(0+)$.


Exercise 4

(By Matt Frito Lundy) For any partition $P$, we have
$$U(P,f) = b-a > 0$$ $$L(P,f) = 0,$$ so $f \notin \mathscr R$.


Exercise 5

(By Matt Frito Lundy) To answer the question Does $f^2 \in \mathscr R$ imply that $f \in \mathscr R$? Consider the function
$$f(x) =
\begin{cases}
1 & x \text{ rational} \\
-1 & x \text{ irrational}.
\end{cases} $$Then $f^2 = 1$ and $f^2 \in \mathscr R$, but $f \notin \mathscr R$.

To answer the question Does $f^3 \in \mathscr R$ imply that $f \in \mathscr R$? Use the fact that $f$ is bounded on $[a,b]$ implies that $f^3$ is bounded on $[a,b]$, so there exists $m, M \in \mathbf R$ such that $m\leq f^3 \leq M$ for all $x \in [a,b]$. Let $\phi (x) = x^{1/3}$, then $\phi$ is continuous on $[m,M]$, and $f = \phi (f^3)$, so $f \in \mathscr R$ on $[a,b]$ by theorem 6.11.

Notice that $\phi(x) = x^{1/2}$ does not work for the first question because $\phi$ is not continuous on $[-1,1]$.


Exercise 6

(By analambanomenos) Following the hint, recall that $P=\cup E_n$ where $E_n$ is a set of $2^n$ disjoint close intervals of length $3^{-n}$ obtained by removing the middle thirds of the intervals in $E_{n-1}$. The total length of the intervals of $E_n$ is $(2/3)^n$ and so $\rightarrow 0$ as $n\rightarrow\infty$. We can replace the closed intervals of $E_n$, $[a_{i,n},b_{i,n}]$ with slightly larger open intervals $(a_{i,n}-\delta/2^n,b_{i,n}+\delta/2^n)$, with total length $(2/3)^n+\delta$, so that we can cover $P$ with a set of disjoint open intervals with total length as small as possible.

Now proceeding as in the proof of Theorem 6.10, let $M=\sup\big|f(x)\big|$, and cover $P$ with a collection of open intervals $(u_j,v_j)$ with total length less than $\varepsilon$. The complement $K$ of the union of the open intervals in $[a,b]$ is compact. Then $f$ is uniformly continuous on $K$ and there exists $\delta>0$ such that $\big|f(s)-f(t)\big|<
\varepsilon$ if $s,t\in K$, $|s-t|<\delta$.

Form a partition $Q=\{x_0,\ldots,x_n\}$ of $[a,b]$ such that each $u_j$ and $v_j$ occurs in $Q$, no point of any segment $(u_j,v_j)$ occurs in $Q$, and if $x_{i-1}$ is not one of the $u_j$, then $\Delta x_i<\delta$.

Note that $M_i-m_i\le 2M$ for every $i$, and that $M_i-m_i\le\varepsilon$ if $x_{i-1}$ is not one of the $u_j$. Hence
\begin{align*}
U(Q,f)-L(Q,f) &= \sum_i (M_i-m_i)\Delta x_i \\
&= \sum_{x_{i-1}\in\{u_j\}}(M_i-m_i)\Delta x_i+\sum_{x_{i-1}\notin\{u_j\}}(M_i-m_i)\Delta x_i \\
&\le 2M\varepsilon + \varepsilon(b-a)=(2M+b-a)\varepsilon.
\end{align*}Hence $f\in\mathscr R$ by Theorem 6.6.


Exercise 7

(By analambanomenos)

(a) By Theorems 6.12(c) and 6.20, $$\lim_{c\rightarrow0}\int_c^1f(x)\,dx=\int_0^1f(x)\,dx-\lim_{c\rightarrow0}\int_0^cf(x)\,dx=\int_0^1f(x)\,dx.$$

(b) Let
$$f(x) =
\begin{cases}
0 & x=0 \\
-\displaystyle\frac{2^{2n-1}}{2n-1} & 2^{-(2n-1)}<x\le2^{-(2n-2)},\;n=1,2,\ldots \\
\displaystyle\frac{2^{2n}}{2n} & 2^{-2n}<x\le2^{-(2n-1)},\;n=1,2,\ldots
\end{cases} $$Then for any positive integer $N$, $$\int_{2^{-N}}^1f(x)\,dx=\sum_{n=1}^N\frac{(-1)^n}{n}$$ which converges as $N\rightarrow\infty$ by Theorem 3.43. However,
$$\int_{2^{-N}}^1\big|f(x)\big|\,dx=\sum_{n=1}^N\frac{1}{n}$$ fails to converge as $N\rightarrow\infty$ by Theorem 3.28.


Exercise 8

(By analambanomenos) For any positive integer $n$ define $g_1(x)=f(n)$ and $g_2(x)=f(n+1)$ for $x\in(n,n+1]$. Since $f$ decreases monotonically, $g_1(x)\ge f(x)\ge g_2(x)$ for all $x\in[1,\infty)$. And since $$\int_n^{n+1}g_1(x)\,dx=f(n)\quad\quad\int_n^{n+1}g_2(x)\,dx=f(n+1)$$ we get by Theorem 6.12(b), for any positive integer $N$, $$\sum_1^Nf(n)=\int_1^Ng_1(x)\,dx\le
\int_1^Nf(x)\,dx\le\int_1^Ng_2(x)\,dx=\sum_2^{N+1}f(n).$$ Hence $\int_1^Nf(x)\,dx$ converges as $N\rightarrow\infty$ if and only if $\sum_1^Nf(n)$ converges. In that case, if $A=\sum_1^\infty f(n)$, then $\int_1^\infty f(x)\,dx$ lies between $A-f(1)$ and $A$.


Exercise 9

(By analambanomenos) If $F$ and $G$ are functions which are differentiable on $[c,1]$ for all $c>0$ and such that $F’=f\in\mathscr R$ and $G’=g\in\mathscr R$ on $[c,1]$ for all $c>0$, then by Theorem 6.22 we have $$\int_c^1F(x)g(x)\,dx=F(1)G(1)-F(c)G(c)-\int_c^1f(x)G(x)\,dx.$$ Suppose that two of the limits $$\lim_{c\rightarrow0}\int_c^1F(x)g(x)\,dx=\int_0^1F(x)g(x)\,dx,\;\;
\lim_{c\rightarrow0}\int_c^1f(x)G(x)\,dx=\int_0^1f(x)G(x)\,dx,\;\;\lim_{c\rightarrow0}F(c)G(c)$$ exist and are finite. Then the third limit exists and is finite, and we have
$$\int_0^1F(x)g(x)\,dx=F(1)G(1)-\lim_{c\rightarrow0}F(c)G(c)-\int_0^1F(x)g(x)\,dx.$$ Similarly, if $F$ and $G$ are functions which are differentiable on $[a,b]$ for all $b>a$ and such that $F’=f\in\mathscr R$ and $G’=g\in\mathscr R$ on $[a,b]$ for all $b>a$, then by Theorem 6.22 we have $$\int_a^bF(x)g(x)\,dx=F(b)G(b)-F(a)G(a)-\int_a^bf(x)G(x)\,dx.$$ Suppose that two of the limits $$\lim_{b\rightarrow\infty}\int_a^bF(x)g(x)\,dx=\int_a^\infty F(x)g(x)\,dx,\;\;\lim_{b\rightarrow\infty}\int_a^bf(x)G(x)\,dx=\int_a^\infty
f(x)G(x)\,dx,\;\;\lim_{b\rightarrow\infty}F(b)G(b)$$ exist and are finite. Then the third limit exists and is finite, and we have $$\int_a^\infty F(x)g(x)\,dx=
\lim_{b\rightarrow\infty}F(b)G(b)-F(a)G(a)-\int_a^\infty F(x)g(x)\,dx.$$

For the example, let $$F(x)=\frac{1}{1+x},\quad F’(x)=f(x)=-\frac{1}{(1+x)^2},\quad G(x)=\sin x,\quad G’(x)=g(x)=\cos x.$$ The functions $F$ and $G$ are differentiable on $[0,b)$, for all $b>0$, and $f\in\mathscr R$, $g\in\mathscr R$ on $[0,b]$ for all $b>0$. Also $$\lim_{b\rightarrow\infty}F(b)G(b)=\lim_{b\rightarrow\infty}\frac{\sin b}{1+b}=0$$ and
$$\lim_{b\rightarrow\infty}\bigg|\int_0^b\frac{\sin x}{(1+x)^2}\,dx\bigg|\le\lim_{b\rightarrow\infty}\int_0^b\frac{|\sin x|}{(1+x)^2}\,dx\le\lim_{b\rightarrow\infty}
\int_0^b\frac{1}{(1+x)^2}\,dx$$ which converges by Exercise 8 since $\sum_0^\infty1/(1+n)^2$ converges. Hence we can apply the results of the first part of this exercise and conclude that $$\int_0^\infty\frac{\cos x}{1+x}\,dx=\lim_{b\rightarrow\infty}\frac{\sin b}{1+b}-\frac{\sin 0}{1+0}+\int_0^\infty\frac{\sin x}{(1+x)^2}\,dx=\int_0^\infty
\frac{\sin x}{(1+x)^2}\,dx.$$We’ve seen above that $\sin x/(1+x)^2$ converges absolutely on $[0,\infty)$. To show that $\cos x/(1+x)$ diverges absolutely,
\begin{align*}
\int_0^\infty\frac{|\cos x|}{1+x}\,dx &= \sum_{k=0}^\infty\int_{2\pi k}^{2\pi(k+1)}\frac{|\cos x|}{1+x}\,dx \\
&\ge\sum_{k=0}^\infty\frac{1}{2\pi(k+1)+1}\int_{2\pi k}^{2\pi(k+1)}|\cos x|\,dx \\
&\ge\sum_{k=0}^\infty\frac{1}{2\pi(k+1)+2\pi}\int_0^{2\pi}|\cos x|\,dx \\
&=\frac{2}{\pi}\sum_{k=0}^\infty\frac{1}{k+2}
\end{align*}a sum which diverges.


Exercise 10

(By analambanomenos) Note that since $q$ and $p=q/(q-1)$ are positive, then $q-1$ is positive.

(a) Fix $u$ and let $f(v)=(u^p/p)+(v^q/q)-uv$. Then $f’(v)=v^{q-1}-u$ and $f”(v)=(q-1)v^{q-2}$ is non-negative for non-negative $v$, so the critical point $v=u^{1/(q-1)}$ is a minimum. Hence
\begin{align*}
\frac{u^p}{p}+\frac{v^q}{q}-uv &\ge \frac{u^p}{p}+\frac{u^{q/(q-1)}}{q}-u^{1+1/(q-1)} \\
&=\biggl(\frac{1}{p}+\frac{1}{q}-1\biggr)u^p \\
&= 0
\end{align*}so that $uv\le(u^p/p)+(v^q/q)$. Equality holds at the critical value $v=u^{1/(q-1)}$, or $v^q=u^{q/(q-1)}=u^p$.

(b) From part (a) we have
\begin{align*}
\int_a^bfg\,d\alpha &\le \int_a^b\biggl(\frac{f^p}{p}+\frac{g^q}{q}\biggr)\,d\alpha \\
&= \frac{1}{p}\int_a^b f^p\,d\alpha+\frac{1}{q}\int_a^bg^q\,d\alpha \\
&= \frac{1}{p}+\frac{1}{q} \\
&= 1.
\end{align*}(c) Define $$A=\biggl(\int_a^b|f|^p\,d\alpha\biggr)^{1/p}\quad\quad B=\biggl(\int_a^b|g|^q\,d\alpha\biggr)^{1/q}.$$ Then
\begin{align*}
\int_a^b\bigg|\frac{f}{A}\bigg|^p\,d\alpha &= \frac{1}{A^p}\int_a^b|f|^p\,d\alpha=1 \\
\int_a^b\bigg|\frac{g}{B}\bigg|^q\,d\alpha &= \frac{1}{B^q}\int_a^b|g|^q\,d\alpha=1.
\end{align*}Applying part (b), we get $$\int_a^b\bigg|\frac{f}{A}\bigg|\cdot\bigg|\frac{g}{B}\bigg|\,d\alpha \le 1,$$ or $$\bigg|\int_a^bfg\,d\alpha\bigg|\le\int_a^b|fg|\,d\alpha\le AB=
\biggl(\int_a^b|f|^p\,d\alpha\biggr)^{1/p}\biggl(\int_a^b|g|^q\,d\alpha\biggr)^{1/q}.$$(d) Let $f\in\mathscr R$, $g\in\mathscr R$ on $[c,1]$ for all $c>0$ such that the improper integrals $\int_0^1|f|^p\,dx$ and $\int_0^1|g|^q\,dx$ exist. Then for all $c>0$ we have
$$\bigg|\int_c^1fg\,dx\bigg|\le\biggl(\int_c^1|f|^p\,dx\biggr)^{1/p}\biggl(\int_c^1|g|^q\,dx\biggr)^{1/q}.$$ Since the right side increases monotonically as $c\rightarrow0$, we can take the limit of both sides to get the desired result.

Similarly, let $f\in\mathscr R$, $g\in\mathscr R$ on $[a,b]$ for all $b>a$ such that the improper integrals $\int_a^\infty|f|^p\,dx$ and $\int_a^\infty|g|^q\,dx$ exist. Then for all $b>a$ we have $$\bigg|\int_a^bfg\,dx\bigg|\le\biggl(\int_a^b|f|^p\,dx\biggr)^{1/p}\biggl(\int_a^b|g|^q\,dx\biggr)^{1/q}.$$ Since the right side increases monotonically as $b\rightarrow\infty$, we can take the limit of both sides to get the desired result.

Baby Rudin 数学分析原理完整第六章习题解答


Linearity

This website is supposed to help you study Linear Algebras. Please only read these solutions after thinking about the problems carefully. Do not just copy these solutions.
Close Menu